Aplicación Teorema de Torricelli (1830) - comentarios Aplicación Teorema de Torricelli 0001 2018-05-07T07:31:18Z https://ejercicios-fyq.com/Aplicacion-Teorema-de-Torricelli#comment9289 2018-05-07T07:31:18Z <p>En este teorema de Torricelli, no queda claro el diametro del orificio por el que saldra el fluido, tambn debe ser importante este punto, ya que dependiendo de eso se quiere saber cuanto liquido sale y a que velocidad</p> Aplicación Teorema de Torricelli 0001 2017-09-01T20:59:27Z https://ejercicios-fyq.com/Aplicacion-Teorema-de-Torricelli#comment7233 2017-09-01T20:59:27Z <p>Yo entiendo con este resultado que h = 0,2 m (según la ley de Torricelli)<br class="autobr"> Me sale<br class="autobr"> a) V = 1,98 m/s<br class="autobr"> b) X = 0,792 m</p> Aplicación Teorema de Torricelli 0001 2017-08-31T16:03:15Z https://ejercicios-fyq.com/Aplicacion-Teorema-de-Torricelli#comment7209 2017-08-31T16:03:15Z <p>no es desde la báse, es desde el nivel de agua, lea bien</p> Aplicación Teorema de Torricelli 0001 2017-07-13T09:29:14Z https://ejercicios-fyq.com/Aplicacion-Teorema-de-Torricelli#comment6903 2017-07-13T09:29:14Z <p>Muchas gracias por tu aporte. Hemos reescrito el enunciado según nos indicas y esperamos que ahora quede más claro el ejercicio y su resolución.</p> Aplicación Teorema de Torricelli 0001 2017-07-10T03:16:22Z https://ejercicios-fyq.com/Aplicacion-Teorema-de-Torricelli#comment6883 2017-07-10T03:16:22Z <p>El problema está mal redactado, debe decir 20 cm por debajo del nivel del líquido y dice 20 cm por encima del recipiente. Por eso es que las personas se confunden</p> Aplicación Teorema de Torricelli 0001 2016-09-11T17:36:30Z https://ejercicios-fyq.com/Aplicacion-Teorema-de-Torricelli#comment4455 2016-09-11T17:36:30Z <p>Entonces la altura h no puede ser de 0.8 metros. La variable h esta definida como la diferencia entre las dos alturas respecto a la superficie del recipiente, esta diferencia sería de 0.2 metros.</p> Aplicación Teorema de Torricelli 0001 2016-04-25T07:20:47Z https://ejercicios-fyq.com/Aplicacion-Teorema-de-Torricelli#comment3099 2016-04-25T07:20:47Z <p>Calcular el chorro de agua del sistema mostrando q si h;? Y ; 7m</p> Aplicación Teorema de Torricelli 0001 2015-11-10T17:30:25Z https://ejercicios-fyq.com/Aplicacion-Teorema-de-Torricelli#comment2069 2015-11-10T17:30:25Z <p>Esta bello el ejercicio te amo</p> Aplicación Teorema de Torricelli 0001 2014-12-03T06:57:11Z https://ejercicios-fyq.com/Aplicacion-Teorema-de-Torricelli#comment1001 2014-12-03T06:57:11Z <p>1. La altura que hay que considerar son los 0,8 m porque el enunciado dice "20 cm por debajo del nivel", y se debe interpretar como el nivel del líquido.</p> <p>2. Tienes razón en que faltaba la raíz cuadrada. Ya está subsanado el error.</p> <p>Gracias.</p> Aplicación Teorema de Torricelli 0001 2014-12-03T06:48:19Z https://ejercicios-fyq.com/Aplicacion-Teorema-de-Torricelli#comment999 2014-12-03T06:48:19Z <p>El inciso b) tiene un dato incorrecto ya que la gota no está a una altura de 0.8 m, sino que a una de 0.2 m, porque el orificio está a 20 cm de la base, no a 80 cm del mismo.<br class="autobr"> También quisiera remarcar que el despeje del tiempo en este mismo inciso necesita una raíz cuadrada ya que el tiempo está elevado al cuadrado, quedando la fórmula:<br class="autobr"> t=(2y/g)^1/2 ("a"^1/2 equivale a la raíz cuadrada)<br class="autobr"> El resultado correcto debería ser<br class="autobr"> *x=0.20 m*</p> Aplicación Teorema de Torricelli 0001 2014-11-07T04:30:11Z https://ejercicios-fyq.com/Aplicacion-Teorema-de-Torricelli#comment852 2014-11-07T04:30:11Z <p>muchas gracias me ayudo mucho</p> Respuesta B 2014-09-24T18:36:38Z https://ejercicios-fyq.com/Aplicacion-Teorema-de-Torricelli#comment788 2014-09-24T18:36:38Z <p>He revisado el problema y he decidido, para que quede más claro (y para poner las soluciones correctas), resolver el problema paso a paso. Espero que te sea de utilidad.</p> <p>Gracias por tu comentario.</p> Respuesta B 2014-09-24T17:14:46Z https://ejercicios-fyq.com/Aplicacion-Teorema-de-Torricelli#comment787 2014-09-24T17:14:46Z <p>he intentado calcular X pero no es el mismo resultado que el que propones aqui si puedes explicame el procedimiento para rectificar gracias</p>